Navigate Test Prep- EMT

Ace your homework & exams now with Quizwiz!

The quickest way to reduce cardiac ischemia in a patient experiencing an acute coronary syndrome is to: A: sit or lay the patient down. B: keep the patient warm. C: elevate the patient's legs. D: give supplemental oxygen.

*A. sit or lay the patient down.* Ischemia is defined as a relative lack of oxygen. In other words, relative to the body's demand for oxygen, its supply is reduced. The quickest way to reduce ischemia in a patient with an acute coronary syndrome, or ACS (eg, unstable angina, acute myocardial infarction) is to reduce the heart's demand for oxygen; simply having the patient cease any exertion and placing him or her in a sitting or lying position will quickly and effectively accomplish this. Although supplemental oxygen increases the oxygen content of the blood and is an important treatment for any patient with ACS, it does not reduce the body's demand for oxygen.

Which of the following actions demonstrates an EMT's knowledge of crime scene preservation? A: Advising a law enforcement officer after moving a coffee table to access a critically injured patient B: Requesting approval from law enforcement before controlling severe bleeding from a patient's arm C: Carefully cutting through the hole in a patient's clothing that was made by a large caliber firearm D: Placing a knife in a plastic zip-lock bag and giving it to a law enforcement officer for safe-keeping

*A. Advising a law enforcement officer after moving a coffee table to access a critically injured patient * After ensuring your own safety, your priority when caring for a patient at a crime scene is to do just that, care for the patient. If you need to move a piece of furniture to gain access to a critically injured patient, move the furniture, treat the patient, and then advise a law enforcement officer of what you moved and where you moved it to. In this way, you are providing immediate care to the patient, but are remaining aware that the location of any obstacles between you and the patient may serve as evidence. Clearly, you are not going to request approval from a law enforcement officer before treating a critically injured patient. Items that may have fingerprints on them, such as knives or guns, should be placed in a paper bag; condensation can accumulate in plastic bags, potentially destroying any evidence. When removing clothing from a gunshot victim, you should make an effort to cut around (not through) the hole in the clothing that was made by the bullet. The hole in a patient's clothing may contain gunshot residue and can provide valuable information regarding the type of weapon used and the distance between the assailant and the victim.

How can you help maximize cardiac output during CPR? A: Allow the chest to fully recoil in between compressions B: Compress the chest at a rate of no more than 100/min C: Ventilate the patient through an advanced airway device D: Deliver rescue breaths until the chest expands widely

*A. Allow the chest to fully recoil in between compressions* Cardiac output is the amount of blood ejected from the left ventricle per minute. Bearing in mind that even the best performed CPR produces only between 25% and 30% of what the patient's cardiac output would otherwise be, there are several actions that you must take to help maximize this. Allowing the chest to fully recoil in between compressions will help draw blood back to the heart; if more blood returns to the heart, more blood can be pumped from the heart with chest compressions. Delivering each rescue breath over a period of 1 second, just enough to produce visible chest rise, will also help maximize cardiac output. If ventilations are given to fast or too forcefully, intrathoracic pressure will increase, resulting in a decrease in the amount of blood that returns to the heart; as a result, cardiac output will decrease. Ventilations are delivered no differently if an advanced airway device (ie, ET tube, multilumen airway, supraglottic airway) has been inserted. Deliver chest compressions at a rate of at least 100 per minute to a depth of at least 2" (at least one third the depth of the chest in infants and children).

A young female is unresponsive after overdosing on an unknown type of drug. Her respirations are slow and shallow and her pulse is slow and weak. Which of the following drugs is the LEAST likely cause of her condition? A: Cocaine B: Valium C: Heroin D: Seconal

*A. Cocaine* Of the drugs listed, cocaine would be the least likely cause of the patient's condition. Cocaine is a central nervous system (CNS) stimulant; you would expect her to be hypertensive, tachycardic, tachypneic, and perhaps even violent. Heroin, Valium, and Seconal are all CNS depressants and could explain her condition. Heroin is an illegal narcotic (opiate), Valium is a benzodiazepine sedative-hypnotic drug, and Seconal is a barbiturate. Narcotics, benzodiazepines, and barbiturates are all CNS depressants. When taken in excess, they cause a decreased level of consciousness, respiratory depression, bradycardia, and hypotension.

Which of the following statements regarding two-rescuer child CPR is correct? A: Compress the chest with one or two hands to a depth that is equal to one third the diameter of the chest B: The chest should not be allowed to fully recoil in between compressions as this may impair venous return C: The chest should be compressed with one hand and a compression to ventilation ratio of 30:2 should be delivered D: A compression to ventilation ratio of 15:2 should be delivered without pauses in compressions to deliver ventilations

*A. Compress the chest with one or two hands to a depth that is equal to one third the diameter of the chest* When performing two-rescuer CPR on a child (1 year of age to the onset of puberty [12 to 14 years of age]), the chest should be compressed with one or two hands (depending on the size of the child), and a compression to ventilation ratio of 15:2 should be delivered. It is important to compress the chest to an adequate depth—one third the anterior-posterior diameter of the chest (about 1 1/2" in the child). The chest should be allowed to fully recoil in between compressions in order to maximize venous return to the heart. If an advanced airway device (ie, ET tube, multilumen airway, supraglottic airway) is not in place, two rescuers should deliver "cycles" of CPR; the compressor should pause briefly so the ventilator can deliver two breaths. A compression to ventilation ratio of 30:2 is used for one-rescuer child CPR. After an advanced airway device has been inserted, "cycles" of CPR should not be performed; compressions should be continuous at a rate of at least 100/min and ventilations should be delivered at a rate of 8 to 10 breaths/min (one breath every 6 to 8 seconds).

Which of the following is a common side effect of nitroglycerin? A: Headache B: Anxiety C: Hypertension D: Nausea

*A. Headache* Because nitroglycerin (NTG) causes vasodilation, including the vessels within the brain, cerebral blood flow increases following the administration of NTG. This often causes a pounding headache for the patient. As uncomfortable as it is for the patient, headaches are a common and expected side effect of the drug. The vasodilatory effects of nitroglycerin could result in hypotension; therefore, the patient's blood pressure should be carefully monitored. Nausea and anxiety are common symptoms of acute coronary syndrome; they are not common side effects of nitroglycerin.

Which of the following is the MOST appropriate response when a patient with chest pain asks you if he or she is having a heart attack? A: I don't know, but we will take good care of you. B: Probably not, but we should transport you to be safe. C: I believe you are, but only a physician can tell for sure. D: Yes, so I recommend going to the hospital.

*A. I don't know, but we will take good care of you.* Patients experiencing chest pain often have a good idea about what is happening, so do not lie or offer false reassurance. Conversely, do not tell the patient they are having a heart attack; this can only be diagnosed by a physician, not an EMT in the field. If asked, "Am I having a heart attack?" an appropriate response would be, "I don't know for sure, but in case you are, we are going to take good care of you."

A 60-year-old man is in cardiac arrest. You begin CPR while your partner applies the AED. What should you do if you receive a no shock message? A: Resume CPR, starting with chest compressions. B: Assess for a carotid pulse for up to 10 seconds. C: Reanalyze his cardiac rhythm after 30 seconds of CPR. D: Ensure that the AED electrodes are properly applied.

*A. Resume CPR, starting with chest compressions.* If the AED gives a no shock advised message, you should immediately resume CPR, starting with chest compressions. After 2 minutes of CPR, reanalyze the patient's cardiac rhythm and follow the AED voice prompts. You should not assess for a pulse if the AED gives a no shock message; this will only cause an unnecessary delay in performing chest compressions. Rarely, if ever, does CPR alone restore a normal cardiac rhythm and pulse. If the AED electrodes are improperly applied, it will not analyze the patient's cardiac rhythm; instead, you will receive a "check patient" or "check electrodes" message. Continue CPR, rhythm analysis every 2 minutes, and defibrillation (if indicated) until ALS personnel arrive or the patient starts to move.

A 60-year-old woman presents with acute respiratory distress. She is conscious and alert, but restless. Her respiratory rate is 26 breaths/min with adequate chest expansion, her breath sounds are clear to auscultation bilaterally, and her oxygen saturation is 90%. Which of the following is the MOST appropriate treatment for this patient? A: Supplemental oxygen with a nonrebreathing mask B: A nasopharyngeal airway and supplemental oxygen C: A nasopharyngeal airway and assisted ventilations D: A nasal cannula with the flowmeter set at 4 to 6 L/min

*A. Supplemental oxygen with a nonrebreathing mask* Although the patient is restless—a sign of hypoxemia—she is conscious and alert and able to maintain her own airway; therefore, an airway adjunct is not needed at this point. Furthermore, her respirations, although increased in rate, are producing adequate tidal volume as evidenced by adequate chest expansion. Therefore, she is not in need of assisted ventilation at this point. Considering her oxygen saturation of 90%, the most appropriate treatment would be to administer high-flow oxygen with a nonrebreathing mask and closely monitor her for signs of inadequate breathing (ie, shallow breaths [reduced tidal volume], decreased level of consciousness, cyanosis). An acutely hypoxemic patient requires more oxygen than a nasal cannula can provide.

Which of the following parameters is the LEAST reliable when assessing the perfusion status of a 2-year-old child? A: Systolic blood pressure B: Skin color and temperature C: Capillary refill time D: Presence of peripheral pulses

*A. Systolic blood pressure* You should never rely on the systolic blood pressure when assessing the perfusion status of anyone. More reliable parameters include assessing peripheral pulses, capillary refill time (most reliable in children younger than 6 years of age), and the condition and temperature of the skin. Remember that the body's compensatory mechanisms work to maintain the blood pressure, so when it falls, this corresponds to decompensated shock.

During your assessment of an unresponsive adult female, you determine that she is apneic. You should: A: assess for a carotid pulse. B: begin chest compressions. C: deliver two rescue breaths. D: place an oropharyngeal airway.

*A. assess for a carotid pulse.* As soon as you determine that an adult patient is apneic or only has agonal gasps, you should assess for a carotid pulse for at least 5 seconds but no more than 10 seconds. If the patient has a pulse, provide rescue breathing at a rate of 10 to 12 breaths/min (one breath every 5 to 6 seconds). If the patient does not have a pulse, perform 30 chest compressions and then open the airway and deliver 2 rescue breaths. When managing a patient who is in cardiac arrest, it is critical to minimize interruptions in chest compressions and to avoid delays in starting chest compressions. After starting CPR, apply the AED as soon as one is available. An airway adjunct should also be inserted as soon as possible.

At the scene of a mass-casualty incident, you notice a bystander who is emotionally upset. An appropriate action to take would be to: A: assign the bystander a simple, non-patient-care task. B: tell the bystander to leave the scene at once. C: have the bystander assist you with patient care. D: notify the police and have the bystander removed.

*A. assign the bystander a simple, non-patient-care task.* One of the most effective ways to reduce stress in a bystander at the scene of a mass-casualty incident is to assign the bystander a task that is not related to patient care. This may involve assisting other bystanders who are having difficulties as well or providing water to the rescuers. An obviously distressed bystander should not simply be sent away from the scene, but should be looked at as a patient as well. Clearly, if the bystander becomes aggressive or violent, law enforcement personnel should get involved.

The two MOST important steps in treating a patient with a contact poisoning are: A: avoiding self-contamination and removing the irritating or corrosive substance from the patient as rapidly as possible. B: determining when the exposure occurred and irrigating the patient's entire body with copious amounts of water. C: immediately washing any dry chemicals off of the patient's skin and removing his or her clothing as soon as possible. D: donning a pair of gloves and performing a physical examination to determine if a significant exposure occurred.

*A. avoiding self-contamination and removing the irritating or corrosive substance from the patient as rapidly as possible.* When caring for a patient with a contact (skin surface) exposure to a poison, the two most important steps in management are avoiding contaminating yourself and removing the substance from the patient's skin as rapidly as possible. The level of personal protective equipment (PPE) you use depends on the type of chemical the patient was exposed to. After ensuring your own safety, remove all of the patient's clothing that has been contaminated, thoroughly brush off any dry chemicals, and then flush the skin with water. Always brush dry chemicals off the skin before irrigating with water; failing to do so may increase the amount of damage caused by the chemical. A physical exam of the patient should be performed only after he or she has been properly decontaminated.

The AED analyzes your pulseless and apneic patient's cardiac rhythm and advises that a shock is NOT indicated. You should: A: resume CPR, starting with chest compressions. B: open the patient's airway and check for breathing. C: reanalyze the cardiac rhythm for positive confirmation. D: assess for a pulse for no more than 10 seconds.

*A. resume CPR, starting with chest compressions.* If the AED advises "no shock," you should immediately resume CPR, starting with chest compressions. Unless the patient starts to move or has other signs of life (ie, coughing), stopping CPR to assess for a pulse should be avoided because it only causes an unnecessary delay in performing chest compressions. The AED is a highly reliable device; if it advises that a shock is or is not indicated, take its word for it! There is no need to reanalyze the patient's cardiac rhythm; this only causes an unnecessary delay in defibrillation (if indicated) and chest compressions.

The MOST effective way to prevent cardiopulmonary arrest in a newborn is to: A: ensure effective oxygenation and ventilation. B: suction its mouth and nose every 3 minutes. C: give blow-by oxygen as soon as it is born. D: perform an Apgar assessment every 5 minutes.

*A. ensure effective oxygenation and ventilation.* Cardiac arrest in the pediatric population, including the newborn, is usually the result of respiratory failure. To prevent this, it is essential to ensure effective oxygenation and ventilation at all times. This involves keeping the airway clear with suction, and, if necessary, administering blow-by oxygen or ventilating with a bag-mask device. Blow-by oxygen is indicated for newborns with central cyanosis that is not rapidly dissipating. The newborn's mouth and nose should be suctioned twice, as soon as its head delivers and after it has been dried, warmed, and properly positioned. Routine suctioning every 3 minutes is not indicated. The need for and extent of resuscitation is determined by assessing the newborn's respiratory effort, heart rate, skin color, and oxygen saturation (SpO2), not the Apgar score. The first Apgar score is not assessed until the newborn is 1 minute of age. Resuscitation, if needed, should begin within 15 to 30 seconds following delivery.

When called to the scene of a structural fire to stand by in case any injuries occur at the scene, you should: A: ensure that your ambulance does not block or hinder other arriving fire apparatus. B: enter the structure with the firefighters so you can provide immediate care to any victims. C: park the ambulance close to the fire so you can rapidly access any patients. D: depart the scene and return to service after the fire has been completely extinguished.

*A. ensure that your ambulance does not block or hinder other arriving fire apparatus.* When standing by at the scene of a structural fire, you must ensure that the ambulance is parked at a safe distance; this will not only keep you safe from the fire itself, but also minimize your risk of injury if the structure collapses. You must also ensure that the ambulance does not block or hinder access to the fire by fire apparatus that arrives after you. You should receive instructions from the fire officer in charge regarding the appropriate location to park the ambulance. It is not the EMT's job, nor is it safe, to enter a structure fire; if victims are located by firefighters, they will be brought to you. Do not depart the scene and return to service until the fire officer in charge releases you; many dangers still exist after the fire has been extinguished.

General guidelines when assessing a 2-year-old child with abdominal pain and adequate perfusion include: A: examining the child in the parent's arms. B: placing the child supine and palpating the abdomen. C: separating the child from the parent to ensure a reliable examination. D: palpating the painful area of the abdomen first.

*A. examining the child in the parent's arms.* If the child's condition is stable, the parent should be allowed to hold the child during the examination. This will minimize anxiety in the child and will make the assessment easier for you. In general, you should avoid separating the child and parents unless the child's condition warrants it. When assessing the abdomen of any patient, you should palpate the most painful area last.

You are responding to a call for a 2-year-old child who fell from a second-story window. With the mechanism of injury and the age of the patient in mind, you should suspect that the primary injury occurred to the child's: A: head. B: lower extremities. C: abdomen. D: chest.

*A. head. * Because a child's head is proportionately larger than the rest of the body when compared to an adult, the head commonly is the primary site of injury. This is especially true in fall-related injuries, in which gravity causes the head to precede the rest of the body. Head injury is a leading cause of traumatic death in infants and small children.

Agonal respirations are not adequate because they are: A: infrequent, gasping respiratory efforts. B: associated with a prolonged inhalation phase. C: the result of an increase in tidal volume. D: characterized by a rapid, irregular pattern.

*A. infrequent, gasping respiratory efforts.* A patient may appear to be breathing after his or her heart has stopped. These occasional, gasping breaths are called agonal respirations (also called agonal gasps) and occur when the respiratory centers in the brain continues to send signals to the respiratory muscles. Agonal respirations are not adequate because they are infrequent, gasping respiratory efforts that produce very little, if any, tidal volume. Patients with agonal respirations require artificial ventilation.

Which of the following assessment parameters is the MOST reliable when determining if a patient with a head injury is improving or deteriorating? A: level of consciousness. B: pupillary reaction. C: rate and depth of breathing. D: systolic blood pressure.

*A. level of consciousness.* All of the options in this question are important parameters to monitor in a patient with a head injury. However, the single most reliable parameter is the patient's level of consciousness (LOC); a person's LOC indicates how the brain is functioning from a global perspective. It should be monitored frequently in order to determine whether the patient's condition is improving (ie, concussion), or deteriorating (ie, intracerebral hemorrhage). In general, level of consciousness serves as the most reliable indicator of perfusion.

You are caring for a 6-year-old child with a swollen, painful deformity to the left forearm. As you communicate with the parents of this child, you should: A: make sure that they remain aware of what you are doing. B: ask them repeatedly how the child was injured. C: use appropriate medical terminology at all times. D: tell them that the child will be transported to the hospital.

*A. make sure that they remain aware of what you are doing.* When caring for any patient, it is important to keep both the patient and family aware of what you are doing. You should avoid medical terminology whenever possible because most laypeople will not understand what you are saying. The plain English approach is much more effective. When caring for children specifically, you should inform the parents of the need for ambulance transportation and why; doing so will provide them with the information necessary to make an informed decision. Asking the parents repeatedly how the child was injured may be construed by some as implying that the child was abused.

After assisting a patient with her epinephrine auto-injector, you should: A: place the device in a puncture-proof container. B: place the device in a red biohazard bag. C: replace the cover and place it in the trash can. D: give it to the patient to have it refilled.

*A. place the device in a puncture-proof container.* After any device is used that has the potential for causing an accidental needle stick or is otherwise contaminated, it should be placed in a puncture-proof container, which usually is red and has a biohazard logo on it. The cover of the auto-injector should never be replaced, nor should a needle be recapped. Epinephrine auto-injectors are not refillable.

The primary clinical feature associated with exposure to a vesicant agent is: A: skin blistering. B: muscle twitching. C: vomiting blood. D: tachycardia.

*A. skin blistering.* The primary route of exposure of blister agents, or vesicants, is the skin. If vesicants are left on the skin or clothing long enough, they produce vapors that can enter the respiratory tract. Vesicants cause burn-like blisters to form on the victim's skin as well as in the respiratory tract (if inhaled). Vesicant agents include sulfur mustard (H), Lewisite (L), and phosgene oxime (CX). The symbols H, L, and CX are military designations. Vesicants usually cause the most damage to damp or moist areas of the body, such as the armpits, groin, and respiratory tract.

While en route to the scene of an injured person, dispatch advises you that law enforcement personnel are at the scene. This indicates that: A: the scene is potentially unsafe. B: the scene is safe for you to enter. C: a crime has been committed. D: the patient is critically injured.

*A. the scene is potentially unsafe.* The presence of law enforcement at the scene indicates, at a minimum, that the scene is potentially unsafe, otherwise they would not be there. While you are en route, you should make radio contact with the police officers at the scene to determine if it is safe for you to enter; you should also inquire about the severity of the patient's injuries. The more information you obtain before you arrive, the better. For all you know, you could arrive and find yourself in the middle of a fire fight! The mere presence of law enforcement at the scene does NOT ensure a safe environment.

Following delivery of a newborn, the 21-year-old mother is experiencing mild vaginal bleeding. You note that her heart rate has increased from 90 to 120 beats/min and she is diaphoretic. In addition to administering high-flow oxygen, treatment should include: A: treating for shock and uterine massage during transport. B: placing her on her left side and transport. C: uterine massage and transport. D: internal vaginal pads and treating for shock during transport.

*A. treating for shock and uterine massage during transport.* Blood loss of up to 500 mL within the first 24 hours after delivery is considered normal and usually is well tolerated by the mother. However, any bleeding, regardless of the severity, with accompanying signs of shock, must be treated accordingly. In this case, you should apply high-flow oxygen, treat the patient for shock by elevating her legs (if allowed by local protocol) and providing warmth, and provide rapid transport to the hospital while massaging the uterine fundus en route. Placing the mother on her left side is appropriate before she delivers and prevents supine hypotensive syndrome. Dressings should never be packed into the vagina; placing pads into the vagina increases the risk for maternal infection.

A middle-aged female with a history of hypertension and high cholesterol complains of chest discomfort. She asks you to take her to the hospital where her personal physician practices, which is 15 miles away. Her blood pressure is 130/70 mm Hg, pulse is 84 beats/min and regular, and respirations are 18 breaths/min and unlabored. Which of the following actions is clearly NOT appropriate for this patient? A: Giving oxygen via nasal cannula B: Allowing her to walk to the ambulance C: Taking her to her choice hospital D: Contacting her physician via phone

*B. Allowing her to walk to the ambulance* You should NEVER allow a patient with a possible cardiac problem to walk to the ambulance. This causes exertion, which increases cardiac oxygen consumption and demand and could worsen his or her condition. Give the patient oxygen in a concentration sufficient to maintain his or her oxygen saturation equal to or greater than 94%. In general, you should transport patients to the hospital of their choice. However, transport to a closer hospital should be considered if you believe the patient is unstable or is at high risk for becoming unstable. If necessary, consult with the patient's physician via phone to determine if he or she thinks the patient should go to a closer hospital.

Which of the following patients is the BEST candidate for an oropharyngeal airway? A: A semiconscious patient who ingested a large quantity of aspirin B: An unresponsive trauma patient with blood draining from the nose C: An unresponsive patient with uncontrolled oropharyngeal bleeding D: Any patient that you suspect of being acutely hypoxemic

*B. An unresponsive trauma patient with blood draining from the nose* The oropharyngeal airway is used to keep the tongue off of the posterior pharynx and is indicated for unresponsive patients without a gag reflex. If an unresponsive patient has severe, uncontrolled oropharyngeal bleeding, your priority is to suction his or her airway in order to prevent aspiration and transport rapidly. Semiconscious patients typically have a gag reflex, although it may be somewhat depressed. Oxygen should be administered to any patient with suspected hypoxemia.

Which of the following actions should be carried out during the primary assessment of an unresponsive patient? A: Obtaining a blood pressure B: Assessing the skin C: Palpating the cranium D: Auscultating the lungs

*B. Assessing the skin* Components of the primary assessment for both responsive and unresponsive patients include assessing and managing the airway and assessing and managing circulation, which includes controlling any major bleeding; assessing the rate, regularity, and quality of the pulse; and assessing the color, condition, and temperature of the skin.

Which of the following injuries is MOST indicative of child abuse? A: Burned hand with splash marks B: Bruising to the upper back C: Multiple bruises to the shins D: Small laceration to the chin

*B. Bruising to the upper back* In order to detect child abuse, you must be familiar with injury locations and patterns consistent with an accident versus those that were intentionally inflicted. It is common for children to trip, fall, and strike their chin or forehead on a solid object; therefore, chin lacerations and hematomas to the forehead are common injuries. Small children frequently hit their legs on coffee tables, resulting in bruises to the shins. If a child accidentally sticks his or her hand in hot water, the hand is quickly pulled back by reflex, resulting in a splash pattern of burns. Injuries found in anatomically unlikely areas, such as the torso (back or front), upper arms and legs, or genitalia, should raise your index of suspicion. Burns that are not accompanied by splash marks should also make you suspicious. For example, if a child's hand or foot is intentionally held in hot water, you will see a clear line of demarcation (stocking-glove effect) without evidence of splash burns.

Which of the following is the MOST common cause of shock in infants and children? A: Severe allergic reaction B: Dehydration C: Accidental poisoning D: Cardiac failure

*B. Dehydration* Common causes of shock in children include infections, dehydration (even from a few episodes of vomiting and/or diarrhea), and blood loss from trauma. Less common causes include severe allergic reactions (anaphylaxis) cardiac failure, and poisonings.

Which of the following statements regarding one-rescuer CPR is correct? A: Ventilations should be delivered over a period of 2 to 3 seconds. B: The chest should be allowed to fully recoil after each compression. C: A compression to ventilation ratio of 15:2 should be delivered. D: You should assess the patient for a pulse after 3 cycles of CPR.

*B. The chest should be allowed to fully recoil after each compression.* When performing CPR on any patient, you should allow the chest to fully recoil after each compression. Incomplete chest recoil causes increased intrathoracic pressure, which may impair blood return to the heart. Assess the patient's pulse after every 5 cycles (about 2 minutes) of CPR; take no longer than 5 to 10 seconds to do this. A compression to ventilation ratio of 30:2 should be performed during all adult and one-rescuer CPR (adult, child, and infant), except for newborns. A compression to ventilation ratio of 15:2 is used during two-rescuer infant and child CPR. Ventilations should be delivered over a period of 1 second each, just enough to produce visible chest rise.

A 5-year-old child in compensated shock secondary to severe vomiting and diarrhea would be expected to have: A: strong, bounding radial pulses. B: a slow capillary refill time. C: a weakly palpable carotid pulse. D: slow, shallow respirations.

*B. a slow capillary refill time.* Compensated shock in the infant or child is characterized by poor peripheral perfusion (eg, delayed [> 2 seconds] capillary refill time, weak peripheral pulses, pallor), tachycardia, and tachypnea. In compensated shock, the child's blood pressure is maintained and his or her mental status is adequate. In decompensated shock, the child's compensatory mechanisms have failed; blood pressure falls, central pulses become weak, and mental status begins to deteriorate. It is critical to recognize an infant or a child in compensated shock, begin immediate treatment, and transport without delay.

The automated external defibrillator (AED) should NOT be used in patients who: A: have a nitroglycerin patch applied to the skin. B: are apneic and have a weak carotid pulse. C: experienced a witnessed cardiac arrest. D: are between 1 and 8 years of age.

*B. are apneic and have a weak carotid pulse.* The AED is only applied to patients in cardiac arrest (eg, pulseless and apneic), whether the arrest was witnessed or unwitnessed. According to the 2010 guidelines for CPR and Emergency Cardiac Care (ECC), AEDs can safely be used in infants and children less than 8 years of age in conjunction with a dose-attenuating system (energy reducer) and pediatric pads. However, if pediatric pads and an energy reducer are unavailable, adult AED pads should be used. A nitroglycerin patch is not a contraindication to the use of an AED; simply remove the patch (with gloved hands) and apply the AED as usual.

In addition to chest pain or discomfort, a patient experiencing an acute coronary syndrome would MOST likely present with: A: profound cyanosis, dry skin, and a headache. B: ashen skin color, diaphoresis, and anxiety. C: severe projectile vomiting and flushed skin. D: irregular breathing and low blood pressure.

*B. ashen skin color, diaphoresis, and anxiety.* Chest pain, pressure, or discomfort is the most common symptom of acute coronary syndrome, or ACS (eg, unstable angina, acute myocardial infarction); it occurs in approximately 80% of cases. Patients with ACS are usually anxious and may have a feeling of impending doom. Nausea and vomiting are common complaints; however, projectile vomiting, which is typically associated with increased intracranial pressure, is uncommon. The skin is often ashen gray and clammy (diaphoretic) because of poor cardiac output and decreased perfusion. Less commonly, the patient's skin is cyanotic. Respirations are usually unlabored unless the patient has congestive heart failure, in which case respirations are rapid and labored; irregular breathing, however, is not common. Blood pressure may fall as a result of decreased cardiac output; however, most patients will have a normal or elevated blood pressure. If the patient complains of a headache, it is usually a side effect of the nitroglycerin they took before your arrival; ACS itself usually does not cause a headache.

A middle-aged male was found unresponsive by his wife. When you arrive at the scene, you assess the patient and determine that he is apneic and pulseless. You should: A: immediately apply the AED, analyze his cardiac rhythm, deliver a shock if indicated, and begin CPR. B: begin CPR starting with chest compressions, apply the AED as soon as possible, and request backup. C: immediately begin CPR, reassess for a carotid pulse after 60 seconds, and then apply the AED. D: perform CPR with a compression-to-ventilation ratio of 15:2, apply the AED, and request backup.

*B. begin CPR starting with chest compressions, apply the AED as soon as possible, and request backup.* As soon as you determine that a patient is unresponsive, pulseless, and apneic, you should begin CPR starting with chest compressions. The appropriate compression-to-ventilation ratio for adult CPR (one- or two-rescuer) is 30:2. A compression-to-ventilation ratio of 15:2 is used for two-rescuer infant and child CPR. Request a backup ambulance as soon as possible; however, do not interrupt CPR to do so. One EMT should perform CPR while the other radios for assistance. Continue CPR and reanalyze the patient's cardiac rhythm every 2 minutes. If indicated, deliver a single shock and immediately resume CPR, starting with chest compressions. If the AED gives a no shock message, resume CPR, starting with chest compressions. Continue CPR, rhythm analysis every 2 minutes, and defibrillation (if indicated), until backup arrives or the patient starts to move.

After clearing the airway of a newborn who is not in distress, it is MOST important for you to: A: apply blow-by oxygen. B: keep the newborn warm. C: clamp and cut the cord. D: obtain an APGAR score.

*B. keep the newborn warm.* After ensuring a patent airway (ie, suctioning and positioning), it is extremely important to keep the newborn warm. Newborns cannot maintain body temperature very well and hypothermia can develop very quickly. Blow-by oxygen should be given if the newborn is breathing adequately, but has cyanosis to the face, neck, or trunk (central cyanosis). The umbilical cord should not be clamped and cut until the cord has stopped pulsating and the newborn is breathing adequately. The Apgar score, which is performed at 1 and 5 minutes after birth (and every 5 minutes thereafter), is not used to determine the need for or extent of resuscitation; respiratory effort, heart rate, skin color, and oxygen saturation (SpO2) are used to determine this.

A 60-year-old man presents with chest pain and difficulty breathing. He is pale, diaphoretic, and in severe pain. As your partner applies supplemental oxygen, you assess his vital signs. His blood pressure is 180/90 mm Hg, pulse is 110 beats/min and irregular, and respirations are 24 breaths/min and labored. You ask him if has taken any nitroglycerin and he tells you that he does not have any but his wife does. You should: A: have him swallow up to four enteric-coated aspirin, continue oxygen therapy, and promptly transport him to the hospital. B: complete your focused physical examination and prepare the patient for immediate transport to an appropriate hospital. C: contact medical control and request permission to assist the patient with up to three doses of his wife's nitroglycerin. D: transport at once, apply the AED in case he develops cardiac arrest, and monitor his vital signs en route to the hospital.

*B. complete your focused physical examination and prepare the patient for immediate transport to an appropriate hospital.* If a patient with suspected cardiac compromise does not have prescribed nitroglycerin (NTG), complete your focused physical examination, continue oxygen therapy, and transport immediately. Do not administer, or request to administer, any medication that is not prescribed to the patient. If the patient develops cardiac arrest, apply the AED and follow its voice prompts. Do not apply the AED to any patient who is not in cardiac arrest. If your protocols allow you to administer aspirin, give up to 325 mg of chewable baby aspirin (have the patient chew the aspirin before swallowing it). Enteric-coated aspirin (aspirin that is coated to prevent stomach upset) is intended to be swallowed without chewing. It takes too long to dissolve, whereas chewable aspirin has a much faster effect.

All of the following would cause an increased level of carbon dioxide in the arterial blood, EXCEPT: A: short exhalation phase. B: deep, rapid breathing. C: slow, shallow breathing. D: reduced tidal volume.

*B. deep, rapid breathing.* Adequate oxygen intake and carbon dioxide elimination require a patent airway and adequate breathing. The level of carbon dioxide in arterial blood can rise for a number of reasons. Reduced tidal volume (shallow depth of breathing) results in insufficient oxygen intake and decreased carbon dioxide elimination. A patient who is breathing slowly (bradypnea) will also experience a decrease in oxygen intake and reduced carbon dioxide elimination. If exhalation is impaired, the body will not eliminate adequate carbon dioxide; therefore, it will accumulate in arterial blood. Deep, rapid breathing (hyperventilation), however, would likely increase carbon dioxide elimination from the body, thus lowering the carbon dioxide content of arterial blood.

The ultimate goal of any EMS quality improvement program is to: A: ensure that all personnel receive an adequate number of continuing education hours. B: deliver a consistently high standard of care to all patients who are encountered. C: recognize all EMTs who demonstrate consistency in providing competent patient care. D: provide protocols to all EMTs and hold them accountable if protocols are not followed.

*B. deliver a consistently high standard of care to all patients who are encountered.* Providing continuing education to all personnel, recognizing those who consistently provide competent patient care, and holding all personnel accountable for adhering to the EMS protocols are all components of any EMS quality improvement program. The ultimate goal, however, is to provide, as a system, a consistently high standard of care to all patients who are encountered.

Your FIRST action in managing a patient with an altered mental status should be to: A: administer high-flow supplemental oxygen. B: determine if the patient is breathing adequately. C: ask a family member how the patient normally acts. D: administer one tube of oral glucose.

*B. determine if the patient is breathing adequately.* When treating a patient with an altered mental status, you must first ensure a patent airway and determine if the patient is breathing adequately. If the patient is breathing adequately, administer supplemental oxygen and continue your assessment. If the patient is not breathing adequately (ie, fast or slow rate, shallow breathing [reduced tidal volume]), assist his or her ventilations. It is important to ask a family member, if available, how the patient normally acts; this will help establish a baseline. Before administering oral glucose, you should assess the patient's blood glucose level to determine if hypoglycemia is the cause of his or her problem and then decide if the patient is able to swallow the glucose, if indicated. If the patient is unable to swallow, do not administer oral glucose, even if he or she is hypoglycemic. Request a paramedic ambulance if possible so the patient can receive intravenous glucose.

When caring for a patient with an acute behavioral crisis, your primary concern should be: A: gathering all of the patient's medications. B: ensuring you and your partner's safety. C: providing safe transport to the hospital. D: obtaining a complete past medical history.

*B. ensuring you and your partner's safety.* When caring for a patient with a behavioral or emotional crisis, your primary concern should be for your own personal safety as well as your partner's safety. Your ultimate goal is to transport the patient to the hospital safely. Gather as much medical history information as possible, but keep in mind that many patients experiencing an emotional or behavioral crisis will not readily provide this information. It is important to remember that patients with emotional or behavioral crises may appear calm initially; however, there is always the potential for them to turn violent.

An unresponsive apneic patient's chest fails to rise after two ventilation attempts. You should: A: reposition the head and reattempt to ventilate. B: immediately proceed to chest compressions. C: suction the airway and reattempt ventilations. D: attempt to ventilate again using more volume.

*B. immediately proceed to chest compressions.* If your initial attempt to ventilate an apneic patient is unsuccessful (that is, you meet resistance or the chest fails to visibly rise), reposition the patient's head and reattempt to ventilate. If the second ventilation is unsuccessful, you should proceed under the assumption that the patient has a severe (complete) airway obstruction. Perform 30 chest compressions, open the airway, and visualize the mouth (remove an object only if you can see it). If you are able to remove the foreign object, attempt to ventilate. If you are not, continue chest compressions. Continue this sequence until the obstruction is relieved or an advanced life support (ALS) ambulance arrives. If ALS response will be delayed, transport, continuing your attempts to relieve the obstruction en route, and coordinate a rendezvous with the ALS unit.

A 22-year-old female fell on her knee and is in severe pain. Her knee is flexed and severely deformed. Her leg is cold to the touch and you are unable to palpate a distal pulse. You should: A: carefully straighten her leg until you restore a distal pulse and then apply padded board splints. B: manually stabilize her injury and contact medical control for further stabilization instructions. C: apply gentle longitudinal traction as you straighten her leg and then apply a traction splint. D: place a pillow behind her knee and stabilize the injury by applying padded board splints.

*B. manually stabilize her injury and contact medical control for further stabilization instructions.* A dislocated knee occurs when the proximal end of the tibia completely displaces from its juncture with the distal femur. In some cases, the popliteal artery behind the knee may be compressed, resulting in compromised distal blood flow. Signs of this include absent distal pulses and a pale extremity that is cool or cold. Manually stabilize the knee and assess for distal pulses. If distal pulses are absent, contact medical control immediately for further stabilization instructions. Medical control may instruct you to make ONE attempt to realign the knee to reduce compression of the popliteal artery and restore distal circulation. If you are unable to restore distal circulation or medical control advises you not to manipulate the injury, splint the knee in the position it was found and transport promptly. Traction splints are contraindicated in any injury to or near the knee.

During the rapid head-to-toe assessment of a patient with multiple injuries, you expose the chest and find an open wound with blood bubbling from it. You should: A: stop your assessment and transport. B: prevent air from entering the wound. C: apply high-flow supplemental oxygen. D: place a porous dressing over the wound.

*B. prevent air from entering the wound.* A sucking chest wound (open pneumothorax) is a life-threatening condition that must be corrected immediately upon discovery. You must take immediate action to prevent air from entering the wound or the patient's condition may continue to deteriorate. Cover a sucking chest wound with an occlusive (non-porous) dressing and secure it on three sides. This will prevent air from entering the pleural space during inhalation. It is important to note, however, that when you cover the wound, you have converted it from an open pneumothorax to a closed pneumothorax. Therefore, you must closely monitor the patient; if signs of a tension pneumothorax develop (ie, worsened respiratory distress, cyanosis, signs of shock), lift the unsecured corner of the dressing to allow air to escape from the pleural space.

A man is experiencing a severe allergic reaction after being stung by a scorpion. He does not have his own epinephrine; however, his wife is allergic to bees and has a prescribed epinephrine auto-injector. You should: A: request an ALS unit to respond to the scene to administer epinephrine. B: provide rapid transport and consider an ALS rendezvous. C: assist the patient with one half the usual dose of the wife's epinephrine. D: assist the patient with the wife's prescribed epinephrine.

*B. provide rapid transport and consider an ALS rendezvous.* If a patient does not have a prescribed epinephrine auto-injector and is experiencing a severe allergic reaction, you should administer supplemental oxygen, assist the patient's ventilations if needed, and transport without delay. Closely monitor the patient's airway and breathing status en route and coordinate an ALS rendezvous if possible. If you carry an epinephrine auto-injector on your ambulance and your protocols allow you to administer it, do so without delay. Otherwise, the ALS unit will be able to administer epinephrine via the intramuscular or intravenous route. Never assist a patient with a medication that is not prescribed to him or her specifically.

You are at the scene where a man panicked while swimming in a small lake. Your initial attempt to rescue him should include: A: rowing a small raft to the victim. B: reaching for the victim with a long stick. C: throwing a rope to the victim. D: swimming to the victim to rescue him.

*B. reaching for the victim with a long stick.* General rules to follow when attempting to rescue a patient from the water include "reach, throw, row, and then go." In this case, you should attempt to reach the victim by having him grab hold of a large stick or similar object. If this is unsuccessful, throw the victim a rope or flotation device (if available). If these are not available, row to the patient in a small raft (if available). Going into the water to retrieve the victim is a last resort. The rescuer must be a strong swimmer because patients who are in danger of drowning are in a state of blind panic and will make every attempt to keep themselves afloat, even if it means forcing the rescuer underwater.

While managing a patient with acute shortness of breath, you attempt to apply a nonrebreathing mask set at 12 L/min. The patient pulls the mask away from his face, stating that it is smothering him. You should: A: securely tape the oxygen mask to the patient's face. B: reassure the patient and apply a nasal cannula instead. C: increase the oxygen flow and reapply the mask. D: inform the patient that refusing oxygen may result in his death.

*B. reassure the patient and apply a nasal cannula instead.* Some adults cannot tolerate the oppressive feeling of an oxygen mask over their face; children are typically less tolerant than adults. You should provide reassurance to the patient and apply a nasal cannula at 2 to 6 L/min, which will likely be better tolerated. Do not force an oxygen mask on a patient's face; doing so will only increase his or her anxiety, which will increase his or her body's oxygen consumption and demand.

You are dispatched to a residence for a 20-year-old male with respiratory distress. When you arrive, you find that the patient has a tracheostomy tube and is ventilator dependent. His mother tells you that he was doing fine, but then suddenly began experiencing breathing difficulty. You should: A: detach the ventilator, suction the tracheostomy tube, and reassess the patient. B: remove him from the mechanical ventilator and ventilate him manually. C: check the settings on the ventilator to ensure that it is functioning properly. D: remove the ventilator tubing and place an oxygen mask over the tracheostomy tube.

*B. remove him from the mechanical ventilator and ventilate him manually.* If a ventilator-dependent patient experiences a sudden onset of respiratory distress, you should first remove him or her from the mechanical ventilator and begin manual ventilation with a bag-mask device; if the patient improves, you will know that the problem was a malfunction with the mechanical ventilator. If the patient does not improve, the tracheostomy tube is likely plugged with thick mucus secretions and requires suctioning. Unless you are familiar with the mechanical ventilator (most EMTs are not), do not attempt to troubleshoot the device by checking the settings; this only wastes time.

When the incident command system is activated at the scene, you should expect to: A: receive instructions and then function independently. B: report back to your section officer in between assignments. C: be assigned one responsibility for the duration of the incident. D: be immediately directed to the established treatment area.

*B. report back to your section officer in between assignments.* The incident command system (ICS) was established in order to maximize effective operations at the scene and maximize the number of lives saved. Deviation from the ICS jeopardizes lives and increases the risk of losing control over the situation. When you arrive at the scene in which the ICS has been activated, you should report to the staging area, where you will be directed to the area in which you are needed. Report to the section officer of that area, receive your instructions, and carry them out. When you have completed your assignment, you must return to the section officer for further instructions. Depending on the situation, you may be sent to another section. The sections that you work in and the responsibilities that you are given may change during the incident. At no time should you attempt to function independently (freelancing); this defeats the purpose of the ICS and puts lives in jeopardy.

You and your partner arrive at the home of a 60-year-old man with shortness of breath. As you enter the residence, you find the patient sitting in his recliner; he is in obvious respiratory distress. As you approach him, he becomes verbally abusive, stating that it took you too long to get to his home. A small handgun is sitting on a table next to his recliner. You should: A: turn around, rapidly exit the residence, and notify law enforcement. B: slowly back your way out of the residence and call law enforcement. C: explain the reason for your delay as your partner administers oxygen. D: verbally distract the patient as your partner attempts to retrieve the gun.

*B. slowly back your way out of the residence and call law enforcement.* There is an immediate threat to you and your partner's safety! The patient is verbally abusive and has a firearm within arm's reach. In this situation, you and your partner should slowly back away from the patient, exit the residence, and notify law enforcement. Do NOT take your eyes off of the patient; if you turn your back, you may take a bullet in the back! Backing away gives you the opportunity to take cover if you see the patient reach for his gun. Remember, your safety comes before all else.

You arrive at the scene shortly after a 55-year-old man collapsed. Two bystanders are performing CPR. Your FIRST action should be to: A: attach the AED and analyze his cardiac rhythm. B: stop CPR so you can assess breathing and pulse. C: check the effectiveness of the CPR in progress. D: insert an oropharyngeal airway and continue CPR

*B. stop CPR so you can assess breathing and pulse.* Upon arriving at a scene where bystander CPR is in progress, you must first confirm that the patient is indeed apneic and pulseless and needs CPR. Bystanders who are not properly trained often perform CPR on patients who do not need it. After confirming cardiac arrest, you should resume CPR and attach the AED as soon as possible.

The appropriate technique for performing two-rescuer CPR on a 4-year-old child includes: A: 30 compressions to 2 ventilations, compressing the chest one third the depth of the chest, and delivering each breath over 1 second. B: 15 compressions to 2 ventilations, compressing the sternum with your thumbs, and delivering at least 100 compressions per minute. C: 15 compressions to 2 ventilations, compressing the sternum with the heel of your hand, and ventilating until visible chest rise occurs. D: 30 compressions to 2 ventilations, compressing the sternum with the heel of both hands, and delivering each breath over 1 to 2 seconds.

*C. 15 compressions to 2 ventilations, compressing the sternum with the heel of your hand, and ventilating until visible chest rise occurs.* When performing two-rescuer CPR on an infant (less than 1 year of age) or a child (1 year of age to the onset of puberty [12 to 14 years of age]), use a compression to ventilation ratio of 15:2. Compress the chest one-third the depth of the chest (about 1 1/2" for infants; about 2" for children), at a rate of at least 100/min, and allow the chest to fully recoil in between compressions. Deliver each breath over 1 second, just enough to produce visible chest rise, and allow complete exhalation. For a child, use the heel of one or both hands to compress the chest, depending on the size of the child. For two-rescuer infant CPR, use the tips of your thumbs to compress the chest (two-thumb, encircling hands technique); the two-finger technique may be used for one-rescuer infant CPR. A 30:2 compression to ventilation ratio is used for all adult and one-rescuer CPR.

When is it MOST appropriate to clamp and cut the umbilical cord? A: After the placenta has completely delivered B: Immediately following delivery of the newborn C: As soon as the cord has stopped pulsating D: Before the newborn has taken its first breath

*C. As soon as the cord has stopped pulsating* Generally, it is safe to clamp and cut the umbilical cord once it has stopped pulsating and the baby is breathing adequately. When blood flow through the umbilical cord ceases, it will stop pulsating; this indicates that the baby is oxygenating its own blood. If the cord does not stop pulsating and/or the baby is not breathing adequately, the cord should not be clamped and cut and the baby should be kept at the level of the mother's perineum and managed appropriately while en route to the hospital.

Which of the following structures is NOT part of the endocrine system? A: Pituitary B: Pancreas C: Gallbladder D: Thyroid

*C. Gallbladder* The gallbladder, which concentrates and stores bile, is not an endocrine organ; it is a digestive organ. Endocrine organs produce hormones, which regulate other body organs and systems. The thyroid regulates metabolism; the pancreas produces insulin and glucagon; and the pituitary gland, which is located within the brain, is the "master" endocrine gland and regulates the function of all endocrine glands in the body.

Which of the following statements regarding crowning is correct? A: Crowning always occurs immediately after the amniotic sac has ruptured. B: Crowning represents the end of the second stage of labor. C: Gentle pressure should be applied to the baby's head during crowning. D: It is safe to transport the patient during crowning if the hospital is close.

*C. Gentle pressure should be applied to the baby's head during crowning.* Crowning occurs when the baby's head is visible at the vaginal opening; it is an obvious sign of delivery in progress. When crowning is observed, you should apply gentle pressure to the infant's head to prevent an explosive delivery. Care must be taken to avoid putting pressure on the fontanelles (the soft spots on the infant's head). Crowning represents the end of the first stage of labor and the beginning of the second stage; it does not always occur immediately after the amniotic sac has ruptured. If the infant's head is born and the amniotic sac is still intact, you need to pinch the thin membrane with your fingers, which will usually cause the sac to easily rupture, and then suction the infant's mouth and nose.

Which of the following is MOST indicative of decompensated shock in a trauma patient with internal bleeding? A: Restlessness B: Tachycardia C: Hypotension D: Clammy skin

*C. Hypotension* In compensated shock, the body is able to maintain blood pressure, usually above 90 to 100 mm Hg, through the physiologic responses of tachycardia and shunting of blood from the skin to the vital organs of the body. Signs of compensated shock include restlessness; pallor; tachycardia; tachypnea; and cool, clammy (diaphoretic) skin. If the signs of compensated shock are not recognized and treatment is delayed, the body's compensatory mechanisms will fail and blood pressure will fall (hypotension). At this point, the patient is said to be in decompensated shock. Do not wait for a trauma patient's blood pressure to fall before initiating treatment; it may be too late.

Which of the following is the preferred initial method for providing artificial ventilations to an apneic adult? A: Two-person bag-valve-mask technique with 100% oxygen B: Flow-restricted, oxygen-powered ventilation device C: Mouth-to-mask technique with supplemental oxygen D: One-person bag-valve-mask technique with 100% oxygen

*C. Mouth-to-mask technique with supplemental oxygen* The preferred initial method for providing artificial ventilations is the mouth-to-mask technique with one-way valve and supplemental oxygen attached. Evidence has show that rescuers who ventilate patients infrequently have difficulty maintaining an adequate seal with a bag-mask device. Because both of the rescuer's hands are freed up when using a pocket face mask, it is easier to maintain an adequate seal, thus providing more effective ventilations. Of course, if two rescuers are available to manage the airway, the two-person bag-mask device technique should be used. The flow-restricted, oxygen-powered ventilation device, also referred to as the manually-triggered ventilator or demand valve, requires an oxygen source to function and would thus not be practical as an initial device for providing artificial ventilations.

A patient presents with severe bradycardia, hypersalivation, vomiting, and excessive tearing. Which of the following agents would MOST likely cause his signs and symptoms? A: Phosgene B: Chlorine C: Soman D: Anthrax

*C. Soman* Nerve agents (eg, V agent [VX], sarin [GB], soman [GD], tabun [GA]) are among the most deadly chemicals developed. Designed to kill large numbers of people with small quantities, nerve agents can cause cardiac arrest within seconds to minutes of exposure. Nerve agents, discovered while in search of a superior pesticide, are in a class of chemical called organophosphates, which are found in household bug sprays, agricultural pesticides, and some industrial chemicals. Organophosphates block an essential enzyme in the nervous system, which cause the body's organs to become overstimulated. The mnemonic "DUMBELS" can help you recall the signs and symptoms of nerve agent exposure; it stands for Diarrhea; Urination; Miosis (constricted pupils); Bradycardia; Emesis (vomiting), Lacrimation (excessive tearing); and Seizures, Salivation, and Sweating. You can also use the mnemonic "SLUDGEM," which stands for Salivation, Lacrimation, Urination, Defecation, GI distress, Emesis, and Miosis.

Assessment and treatment of a patient with severe abdominal pain should include: A: vigorously palpating the abdomen. B: giving the patient small sips of water. C: administering supplemental oxygen. D: placing the patient in a supine position.

*C. administering supplemental oxygen.* Patients with severe abdominal pain should be given supplemental oxygen (in a concentration sufficient to maintain an SpO2 of greater than 94%) and immediate transport. Most patients with abdominal pain prefer to lie on their side with their knees drawn up into their chest (fetal position), which takes pressure off of the abdominal musculature and often affords them some relief. Vigorous palpation of the abdomen should be avoided as this will only cause the patient more pain; instead, gently palpate each of the four abdominal quadrants, palpating the most painful area last. Because the patient may become nauseated and vomit and the possibility that he or she may require surgery, do not give him or her anything to eat or drink.

You and your partner are performing CPR on a 2-year-old female in cardiac arrest. During your resuscitation attempt, you should: A: hyperventilate her because she is severely hypoxic. B: attach the AED pads after 5 minutes of high-quality CPR. C: allow the chest to fully recoil in between compressions. D: perform compressions and ventilations at a ratio of 30:2.

*C. allow the chest to fully recoil in between compressions.* When performing two-rescuer CPR on an infant or a child, use a compression to ventilation ratio of 15:2 (30:2 for one-rescuer infant or child CPR), compress the chest one-third the depth of the chest (about 2"), and allow the chest to fully recoil between compressions. Full recoil of the chest is essential to high-quality CPR; it maximizes the amount of blood that returns to the heart, which maximizes the amount of blood ejected from the left ventricle during chest compressions. Do not hyperventilate any patient; deliver each breath over 1 second while observing the chest for visible rise. Hyperventilation causes gastric distention and increases the risk of aspiration if regurgitation occurs. Furthermore, hyperventilation causes a reduction in blood return to the heart because it hyperinflates the lungs and puts pressure on the heart. Attach pediatric AED pads as soon as possible, analyze the child's cardiac rhythm, and deliver a single shock if indicated.

A behavioral crisis is MOST accurately defined as: A: a chronic mental health problem in which the patient experiences frequent thoughts of suicide or other self-destructive behavior. B: a persistent feeling of sadness, despair, or hopelessness that incapacitates the patient and prevents him or her from interacting socially. C: any reaction to an event that interferes with the activities of daily living or has become unacceptable to the patient, family, or community. D: an acute psychiatric emergency characterized by violent behavior, mood swings, and a loss of connection to reality.

*C. any reaction to an event that interferes with the activities of daily living or has become unacceptable to the patient, family, or community.* The definition of a behavioral crisis or emergency is any reaction to an event that interferes with the activities of daily living (eg, bathing, dressing, eating) or has become unacceptable to the patient, his or her family, or the general community. Some patients react to an event with violent behavior or suicidal thoughts; others react with depression. Regardless of the reaction, it significantly interferes with the patient's life or is unacceptable to his or her family and/or the community.

The safest emergency vehicle operator is one who: A: has a positive attitude. B: drives with lights and siren. C: drives with due regard. D: is physically fit.

*C. drives with due regard.* One of the most important attributes of a safe emergency vehicle operator is the ability to drive with due regard for others. This means that the operator must be aware of others around him or her and to keep their safety in mind. The EMT should never assume that all drivers will see or hear the ambulance. A positive attitude about one's ability to safely operate an emergency vehicle is also an important attribute. Although sometimes indicated for the patient's condition, the use of lights and siren increase the risk of an ambulance crash.

When using the power lift to lift a stretcher, you should: A: place your hands palms down on the stretcher. B: bend at the waist and keep your back straight. C: ensure that you lift with your palms facing up. D: maintain a slight inward curve to your back.

*C. ensure that you lift with your palms facing up.* In order to achieve the best grip and to avoid injury to your wrists, you should lift a stretcher, backboard, or other carrying device with your palms facing up. Do not bend at the waist; bend at the knees and keep your back in a straight, locked-in position.

Hypoxia-induced unresponsiveness during a submersion injury is usually the result of: A: associated hypothermia. B: water in the lungs. C: laryngospasm. D: a cardiac dysrhythmia.

*C. laryngospasm.* When a patient falls into the water or becomes panicked when in the water, he or she begins to swallow large amounts of water. Even a small amount of water near the larynx can cause a spasm, which closes off the airway. This results in hypoxia, loss of consciousness, and cardiac dysrhythmias. If the patient is not removed from the water at once and treated aggressively, hypoxia and acidosis will eventually result in cardiac arrest.

Following an apparent febrile seizure, a 4-year-old boy is alert and crying. His skin is hot and moist. Appropriate treatment this child includes: A: rapidly cooling the child in cold water. B: keeping the child warm and providing transport. C: offering oxygen and providing transport. D: allowing the parents to transport the child.

*C. offering oxygen and providing transport.* Most children with febrile seizures do not have any permanent aftereffects. The most appropriate treatment is to offer the child oxygen (usually via the blow-by technique), allow a parent to accompany the child in the back of the ambulance, and transport to the hospital. Although most seizures in children result from a simple infection (ie, ear infection) that causes an abrupt rise in body temperature, other illnesses such as meningitis and encephalitis can cause seizures as well and are far more serious. For this reason, any child with fever and seizures should be evaluated in the emergency department. Rapid cooling of the child should be avoided as this will likely cause shivering, which could abruptly increase the child's temperature and cause another seizure. Children with a fever should be kept cool during transport (ie, removing clothing), but not to the point where they shiver.

You arrive at the scene of a 56-year-old man who collapsed. The patient's wife tells you that he suddenly grabbed his chest and then passed out. Your assessment reveals that he is apneic and pulseless. As your partner begins one-rescuer CPR, you should: A: insert an airway adjunct. B: notify medical control. C: prepare the AED for use. D: obtain a SAMPLE history.

*C. prepare the AED for use.* Immediate treatment for a patient in cardiac arrest involves performing CPR and applying the AED as soon as possible. After applying the AED pads to the patient's chest (around your partner's compressing hands), analyze his cardiac rhythm, deliver a shock if indicated, and immediately resume CPR (starting with chest compressions). Management of the airway, including insertion of an airway adjunct, should occur during the 2-minute period of CPR in between cardiac rhythm analysis and defibrillation. While CPR is in progress, obtain as much of the patient's medical history from his wife as possible, and notify medical control when it is practical (ie, you have more help at the scene).

A 50-year-old woman with a history of epilepsy is actively seizing. Care for this patient should focus primarily on: A: placing a bite block in between her molars and giving her oxygen. B: administering high-flow oxygen and requesting an ALS ambulance. C: protecting her from injury and ensuring adequate ventilation. D: frequently suctioning her airway and carefully restraining her.

*C. protecting her from injury and ensuring adequate ventilation.* Seizure deaths are most frequently the result of hypoxia. When a person is actively seizing, he or she is not breathing adequately. Your primary focus when treating a seizure patient is to protect him or her from injury and to ensure adequate ventilation and oxygenation. Many seizing patients require assisted ventilation. Suction the oropharynx only if the patient has secretions in his or her mouth. Do NOT insert anything into the mouth of a seizing patient; doing so may cause an airway obstruction or damage the soft tissues of the mouth, resulting in bleeding. Do not attempt to restrain an actively seizing patient; doing so may result in musculoskeletal injuries. Request an ALS ambulance per your local protocols.

You should suspect physical abuse of a 4-year-old child if you observe: A: curious siblings who are watching you. B: that the child clings to his or her parent. C: purple and yellow bruises to the thighs. D: bruises to the anterior tibial area.

*C. purple and yellow bruises to the thighs.* EMTs always must keep the possibility of child abuse in the back of their minds when dealing with an injured child. Signs that would indicate abuse include, but are not limited to, bruises in areas that are not likely to be injured, such as the thigh, back, and chest; multiple bruises in varying colors, indicating various stages of healing; conflicting stories among caregivers; injuries that are beyond the developmental abilities of the child, such as a 2-year-old child who has "fallen from her bicycle"; and cases in which the child does not look at the parents or cling to them as one would expect an injured child to do. Siblings of an abused child are typically not curious onlookers as they have become accustomed to the abusive environment. As an EMT, you have a legal obligation to report any and all cases of suspected child abuse. Never accuse anyone of abusing a child. If you are wrong, you could be held liable for slander.

A woman stabbed her boyfriend in the cheek with a dinner fork during an argument. Police have the woman in custody. The patient still has the fork impaled in his cheek. He is conscious and alert, breathing adequately, and has blood in his oropharynx. You should: A: apply high-flow oxygen via a nonrebreathing mask, carefully remove the fork, and control any external bleeding. B: suction his oropharynx, carefully cut the fork to make it shorter, control any external bleeding, and secure the fork in place. C: suction his oropharynx, control any external bleeding, stabilize the fork in place, and protect it with bulky dressings. D: carefully remove the fork, suction his oropharynx as needed, and pack the inside of his cheek with sterile gauze pads.

*C. suction his oropharynx, control any external bleeding, stabilize the fork in place, and protect it with bulky dressings.* An impaled object in the cheek should be removed if it interferes with your ability to manage the patient's airway. In this case, however, the patient is breathing adequately and does not require aggressive airway care (eg, ventilatory assistance). The most practical approach is to suction the blood from his oropharynx, which will prevent him from swallowing it, vomiting it, and aspirating it. Stabilize the fork in place and protect it with bulky dressings; removing an impaled object from the cheek in the opposite direction it entered may cause further soft tissue damage. Transport the patient in a sitting position and suction his oropharynx en route as needed. There is no reason to cut the fork to make it shorter; this will only unnecessarily manipulate it, potentially causing further soft tissue damage and increased bleeding.

Ischemic heart disease is a condition in which: A: a portion of the heart muscle dies because of a prolonged lack of oxygen. B: an acute event leads to a significant decrease in the pumping force of the heart. C: there is a decrease in blood flow to one or more portions of the heart muscle. D: the coronary arteries dilate, thus preventing effective blood flow to the heart.

*C. there is a decrease in blood flow to one or more portions of the heart muscle.* Chest pain or discomfort that is related to the heart usually stems from a condition called ischemia (insufficient oxygen). Because of a partial or complete blockage of blood flow through one or more coronary arteries, the tissue of the heart muscle (myocardium) fails to get enough oxygen and nutrients relative to its needs. Therefore, ischemic heart disease is a condition involving a decrease in blood flow, and therefore oxygen, to one or more portions of the myocardium. If blood flow to the ischemic portion of the myocardium is not restored, it eventually dies (myocardial infarction). Dilation of the coronary arteries increases, not decreases, blood flow to the heart. If an event such as a myocardial infarction damages the heart and significantly decreases its ability to contract forcefully, heart failure may occur.

The 5-minute Apgar assessment of a newborn reveals a heart rate of 130 beats/min, cyanosis to the hands and feet, and rapid respirations. The infant cries when you flick the soles of its feet and resists attempts to straighten its legs. These findings equate to an Apgar score of: A: 10 B: 7 C: 8 D: 9

*D. 9* The Apgar score, which is obtained at 1 and 5 minutes after birth (and every 5 minutes thereafter), assigns numbers (0, 1, or 2) to the following five areas: Appearance, Pulse, Grimace, Activity, and Respirations. A score of 1 is assigned for appearance if the newborn's body is pink, but its hands and feet remain blue. If its heart rate is greater than 100 beats/min, it receives a score of 2 for the pulse. If it cries and tries to move its foot away when soles of its feet are flicked, it is assigned a score of 2 for grimace/irritability. If it resists attempts to straighten its hips and knees, a score of 2 is assigned for activity/muscle tone. If its respirations are rapid, a score of 2 is assigned. Based on these parameters, the newborn in this scenario would receive an Apgar score of 9. Refer to your EMT textbook for a complete review of the Apgar score.

Which of the following patients with diabetes is the BEST candidate for oral glucose? A: A semiconscious patient with pale, clammy skin B: An unresponsive patient who took too much insulin C: A confused patient with suspected hyperglycemia D: A confused patient who has cool, clammy skin

*D. A confused patient who has cool, clammy skin* Oral glucose is indicated for patients with diabetes who are in insulin shock (hypoglycemic crisis), or for any patient with symptomatic hypoglycemia (as documented by glucometer). The patient must be conscious and alert enough to be able to swallow the glucose, which comes in a tube of gel. If the patient is unresponsive or otherwise unable to swallow the glucose, you should provide rapid transport, providing the appropriate airway management en route, and consider an ALS intercept.

In which of the following situations would the EMT MOST likely deliver a baby at the scene? A: Contractions are 8 to 10 minutes apart and irregular. B: The hospital is 15 miles away and crowning is not present. C: The amniotic sac has ruptured and contractions occur regularly. D: A tornado has struck and blocked the only route to the hospital.

*D. A tornado has struck and blocked the only route to the hospital.* The EMT should prepare for delivery at the scene when delivery is imminent or can be expected within a few minutes (eg, the mother feels the urge to push, contractions are 2 to 3 minutes apart and regular, crowning is present); when a natural disaster (eg, flood, fire, tornado) has occurred and your route to the hospital is blocked; and when transportation is not available (eg, mechanical malfunction of the ambulance). These are just some of the factors to consider when making that critical decision. Rupture of the amniotic sac and the onset of contractions signals the beginning of the first stage of labor; delivery is usually not imminent at this point and it is generally safe to transport.

Which of the following is a physiologic effect of epinephrine when used to treat anaphylactic shock? A: As a vasoconstrictor, it lowers the blood pressure. B: As an antihistamine, it blocks chemicals that cause the reaction. C: As a vasodilator, it increases the blood pressure. D: As a bronchodilator, it improves the patient's breathing.

*D. As a bronchodilator, it improves the patient's breathing.* Epinephrine possesses dual effects. As a bronchodilator, it relaxes the smooth muscle of the bronchioles and improves the patient's breathing. As a vasoconstrictor, it constricts the blood vessels and increases the patient's blood pressure. Diphenhydramine (Benadryl) is an antihistamine; it blocks H1 histamine receptor sites, which blocks the release of the chemicals (histamines) that are causing the allergic reaction.

Who has ultimate authority for all issues regarding patient care at the scene of a mass-casualty incident? A: Treatment officer B: Most experienced EMT C: Incident commander D: EMS medical director

*D. EMS medical director* The incident commander (IC) is responsible for all logistical and operational aspects of a mass-casualty incident (MCI), such as designating section officers and working in conjunction with other agencies (eg, police, fire, EMS). For all issues regarding patient care, the EMS medical director has ultimate authority. Although the treatment officer is responsible for overseeing all emergency care provided at the scene, and EMTs working in the treatment area are providing direct patient care, these personnel are still functioning under the physician's license. During an MCI, the IC (or his or her designee) is typically in contact with the medical director, who is located at the base station hospital, via mobile phone or two-way radio. In some cases, the medical director may be physically present at the incident.

Which of the following is an abnormal finding? A: Rapid, irregular breathing in a newly born infant B: Systolic BP of 100 mm Hg in a 10-year-old child C: Respiratory rate of 26 breaths/min in a 2-year-old child D: Heart rate of 80 beats/min in a 3-month-old infant

*D. Heart rate of 80 beats/min in a 3-month-old infant* An infant between 1 month and 1 year of age typically has a heart rate between 100 and 160 beats/min; a heart rate less than 100 beats/min in this age group is considered to be bradycardic. Newborn infants normally have irregular breathing that ranges between 40 and 60 breaths/min. The systolic BP for a child between 6 and 12 years of age typically ranges between 80 and 110 mm Hg. The respiratory rate in a child between 1 and 3 years of age typically ranges between 20 and 30 breaths/min.

Which of the following statements regarding ventricular fibrillation (V-Fib) is correct? A: Patients in V-Fib should be defibrillated after every 60 seconds of CPR. B: Any patient in V-Fib must receive CPR for 2 minutes prior to defibrillation. C: Loss of consciousness occurs within minutes after the onset of V-Fib. D: In V-Fib, the heart is not pumping any blood and the patient is pulseless.

*D. In V-Fib, the heart is not pumping any blood and the patient is pulseless.* Ventricular fibrillation (V-Fib) is a disorganized, ineffective quivering of the heart muscle. No blood is pumped through the body and the patient is pulseless. Loss of consciousness occurs within seconds following the onset of V-Fib. Patients in V-Fib are treated with high-quality CPR and defibrillation every 2 minutes if needed. When treating a patient in cardiac arrest, whether the patient's arrest was witnessed or unwitnessed, begin immediate CPR and apply the AED as soon as it is available.

Which of the following devices is contraindicated in patients with blunt chest trauma? A: Nasal airway B: Bag-mask device C: Oral airway D: Oxygen-powered ventilator

*D. Oxygen-powered ventilator* The flow-restricted, oxygen-powered ventilation device (FROPVD), also referred to as an oxygen-powered ventilator or manually-triggered ventilator, should not be used in patients with chest trauma; it delivers oxygen under high pressure (40 L/min) and may worsen the patient's injury. The FROPVD is also associated with a high incidence of gastric distention. The FROPVD is also contraindicated in pediatric patients and in patients with COPD. Infants and children have small lungs; the high ventilatory pressure delivered by the FROPVD can easily cause a pneumothorax. Patients with COPD often have air trapped in their lungs; excessive ventilatory pressure may cause alveolar rupture or a pneumothorax.

Which of the following is MOST indicative of compensated shock in an adult? A: Weak carotid pulse, cool skin, increased respiratory rate B: Unresponsive, pallor, absent radial pulses, tachypnea C: Confusion, mottling, tachycardia, BP of 88/60 mm Hg D: Restless, diaphoresis, tachypnea, BP of 104/64 mm Hg

*D. Restless, diaphoresis, tachypnea, BP of 104/64 mm Hg* In compensated shock, the nervous system is mounting a physiologic response to an underlying illness or injury in order to maintain perfusion to vital organs such as the brain, heart, and kidneys. The patient with compensated shock is restless or anxious, has poor peripheral perfusion (eg, pallor, diaphoresis), tachycardia, and increased respirations (tachypnea). However, his or her blood pressure is maintained, usually above 90 to 100 mm Hg. In decompensated shock, the body's compensatory mechanisms fail, blood pressure begins to fall, and perfusion to vital organs decreases. Other signs of decompensated shock include a decreased level of consciousness, absent peripheral pulses (radial), and weak central pulses (carotid, femoral).

Which of the following would MOST likely cause a rapid drop in a patient's blood glucose level? A: Mild exertion after eating a meal B: Eating a meal after taking insulin C: Forgetting to take prescribed insulin D: Taking too much prescribed insulin

*D. Taking too much prescribed insulin* Compared to diabetic coma (hyperglycemic ketoacidosis, hyperglycemic crisis), insulin shock (hypoglycemia, hypoglycemic crisis) has a rapid onset. It is commonly caused when a patient accidentally takes too much prescribed insulin. Insulin is a fast-acting drug that rapidly causes glucose to exit the bloodstream and enter the cell. Other common causes of hypoglycemia include taking a regular dose of insulin but not eating or taking insulin and exercising heavily. Eating a meal after taking insulin typically does not cause a significant change in the patient's blood glucose level. If a person fails to take his or her insulin, glucose will not be able to enter the cell and will accumulate in the bloodstream (hyperglycemia).

Which of the following statements regarding lightning-related injuries is correct? A: Multiple fractures, including those of the cervical spine, are the most common cause of lightning-related deaths. B: The majority of patients who are struck by lightning die, even if CPR is provided immediately. C: Because of the high electrical energy associated with lightning, full-thickness burns are a common finding. D: The cardiovascular and nervous systems are most commonly injured during a lightning strike.

*D. The cardiovascular and nervous systems are most commonly injured during a lightning strike.* The cardiovascular and nervous systems are most commonly injured during a lightning strike; therefore, respiratory or cardiac arrest is the most common cause of lightning-related deaths, especially if CPR is delayed. A lightning strike typically induces asystole (cardiac standstill); however, this spontaneously resolves in some people or can be reversed with early, high-quality CPR. The energy associated with lightning is composed of direct current (DC) of up to 200,000 amperes and a potential of 100 million volts or more; however, the duration of a lightning strike is short. Therefore, skin burns are usually superficial; full-thickness (third-degree) burns are rare. Lightning causes massive contraction of all the body's muscles, potentially resulting in long bone and spinal fractures. Although this can clearly increase morbidity, it is not the most common cause of lightning-related deaths.

Aspirin is beneficial to patients experiencing an acute coronary syndrome because it: A: effectively relieves their chest pain. B: decreases cardiac workload by lowering the BP. C: destroys the clot that is blocking a coronary artery. D: prevents a clot from getting larger.

*D. prevents a clot from getting larger.* Early administration of baby aspirin (160 to 325 mg) to patients with acute coronary syndrome (ACS) has clearly been shown to reduce mortality and morbidity. Aspirin (acetylsalicylic acid [ASA]) prevents the clot in a coronary artery from getting larger by inhibiting platelet aggregation; in other words, it makes the platelets less sticky, which means that they will have less of a tendency to clump together. Aspirin does not relieve the chest pain or discomfort associated with ACS, nor does it reduce blood pressure. Furthermore, aspirin does not destroy the clot that is blocking a coronary artery; fibrinolytic (clot-buster) drugs actually destroy the clot.

An elderly woman with COPD presents with a decreased level of consciousness, cyanosis to her face and neck, and labored respirations. Her pulse is rapid and weak and her oxygen saturation is 76%. You should: A: apply oxygen via nasal cannula and reassess her respiratory status. B: avoid high-flow oxygen because this may cause her to stop breathing. C: insert a nasal airway and give her oxygen via a nonrebreathing mask. D: assist her ventilations with a bag-mask device and high-flow oxygen.

*D. assist her ventilations with a bag-mask device and high-flow oxygen.* The patient in this scenario is experiencing an exacerbation (worsening) of her COPD. Her decreased level of consciousness; cyanosis; weak, rapid pulse; low oxygen saturation (SpO2); and labored breathing clearly indicate that she is not breathing adequately. Therefore, you should assist her ventilations with a bag-mask device and high-flow oxygen; if you don't, she will continue to deteriorate, possibly to the point of cardiac arrest. If needed, insert a nasal airway adjunct to help keep her airway open. Regardless of the patient's history of COPD, you must NOT withhold oxygen from her. Respiratory depression in COPD patients who receive high-flow oxygen is highly uncommon. Death due to hypoxia, however, is very common.

Your partner has applied the AED to a cardiac arrest patient and has received a shock advised message. While the AED is charging, you should: A: cease all contact with the patient until the AED has delivered the shock. B: perform rescue breathing only until the AED is charged and ready to shock. C: retrieve the airway equipment and prepare to ventilate the patient. D: continue chest compressions until your partner tells you to stand clear.

*D. continue chest compressions until your partner tells you to stand clear.* It is important to minimize interruptions in CPR, especially chest compressions, when at all possible. All contact with the patient must cease while the AED is analyzing. However, if the AED gives a shock advised message and begins charging, you should resume chest compressions until the AED is charged and ready to deliver the shock; at this point, you should cease contact with the patient. As soon as the AED delivers the shock, resume CPR starting with chest compressions.

A 70-year-old man with a history of emphysema and congestive heart failure is in cardiac arrest. His wife tells you that he collapsed about 5 minutes before your arrival. Your partner begins one-rescuer CPR as you prepare the AED. As you are applying the AED pads, the man's wife tells you that she wants you to let him die in peace. You should: A: cease resuscitation only if the AED does not indicate a shock. B: perform rescue breathing only and contact medical control. C: comply with her request and cease all resuscitative efforts. D: continue performing CPR and ask her if he has a living will.

*D. continue performing CPR and ask her if he has a living will.* When faced with a situation in which a family member does not wish for you to attempt resuscitation of a loved one, you should inquire about the presence of a living will or out-of-hospital do not attempt resuscitation (OOH-DNAR) order. If a valid living will or OOH-DNAR order is produced, it is generally acceptable to cease resuscitative efforts; consult medical control as needed. If a valid living will or OOH-DNAR order is not available, the most prudent action would be to continue CPR and contact medical control. Even in the absence of such documentation, medical control may advise you to cease resuscitation based on the wishes of the family and the patient's medical history. When in doubt, err on the side of the patient and attempt resuscitation. Few would argue that it is preferable to defend why resuscitation was attempted as opposed to why it was not.

Signs of inadequate breathing in an unresponsive patient include: A: warm, moist skin. B: symmetrical chest rise. C: an irregular pulse. D: cyanotic oral mucosa.

*D. cyanotic oral mucosa.* Signs of inadequate breathing in both responsive and unresponsive patients include a respiratory rate that is too slow (less than 12 breaths/min) or too fast (greater than 20 breaths/min); shallow (reduced tidal volume), irregular, or gasping respirations; asymmetrical (unequal) chest rise; abnormal respiratory sounds, such as wheezing, stridor, or gurgling; and abnormal skin color and condition (ie, cool or cold skin, pallor, diaphoresis, cyanosis). An irregular pulse indicates a cardiac dysrhythmia.

After an adult cardiac arrest patient has been intubated by a paramedic, you are providing ventilations as your partner performs chest compressions. When ventilating the patient, you should: A: deliver each breath over 2 seconds at a rate of 12 to 15 breaths/min. B: hyperventilate the patient to maximize carbon dioxide elimination. C: deliver 2 breaths during a brief pause in chest compressions. D: deliver each breath over 1 second at a rate of 8 to 10 breaths/min.

*D. deliver each breath over 1 second at a rate of 8 to 10 breaths/min.* When ventilating an adult cardiac arrest patient with an advanced airway in place (ie, ET tube, multilumen airway, supraglottic airway), you should deliver each breath over a period of 1 second—just enough to produce visible chest rise—at a rate of 8 to 10 breaths/min (one breath every 6 to 8 seconds). Do not attempt to synchronize ventilations with chest compressions once the airway has been secured with an advanced device. Hyperventilation should be avoided as it may result in increased intrathoracic pressure, decreased blood return to the heart, and as a result, less effective chest compressions.

While caring for a trauma patient, blood splashes into an EMT's eyes. This is an example of: A: indirect contact. B: infection. C: transmission. D: exposure.

*D. exposure.* Exposure occurs when an individual comes in direct contact with blood or other bodily fluids. Examples of direct exposure include blood splashing into the eyes or mouth and an accidental stick with a contaminated needle or other sharp object. Exposure does not always result in disease transmission and subsequent infection; that depends on whether the patient has an infectious disease. Routine use of standard precautions will afford the EMT the best protection from exposure to an infectious disease.

As you begin ventilating an unresponsive apneic man, you hear gurgling in his upper airway. Your MOST immediate action should be to: A: squeeze the bag-mask device with less force and reassess. B: reposition the patient's airway and continue ventilations. C: suction the patient's airway for no longer than 15 seconds. D: quickly turn the patient onto his side so secretions can drain.

*D. quickly turn the patient onto his side so secretions can drain.* Gurgling in the airway indicates the presence of vomitus or other secretions. If this is noted, you should immediately turn the patient onto his side to allow the secretions to drain. After placing the patient on his side, suction his airway for no longer than 15 seconds. To continue ventilating a patient whose airway is full of vomitus or secretions will force the secretions into the trachea, resulting in aspiration. Aspiration significantly increases mortality!

You are assessing a young male who was stabbed in the right lower chest. He is semiconscious and has labored breathing, collapsed jugular veins, and absent breath sounds on the right side of his chest. This patient MOST likely has a: A: liver laceration. B: ruptured spleen. C: pneumothorax. D: hemothorax.

*D. hemothorax.* You should suspect a hemothorax if a patient with chest trauma presents with shock, especially if the injury was caused by penetrating trauma. Hemothorax occurs when blood collects in the pleural space and compresses the lung, resulting in shock and respiratory compromise. Other signs include collapsed jugular veins (due to low blood volume), labored breathing, and decreased or absent breath sounds on the side of the injury. A pneumothorax (air in the pleural space) is also associated with difficulty breathing and unilaterally decreased or absent breath sounds; however, the jugular veins are usually not collapsed. If excessive air accumulates within the pleural space, however, pressure will shift across the mediastinum and affect the uninjured lung (tension pneumothorax); if this occurs, the jugular veins may become engorged (distended). Splenic injury is unlikely; the patient's injury is on the right side and the spleen is on the left. A liver laceration can cause severe shock; however, it is not associated with unilaterally decreased breath sounds or labored breathing.

The function of the uterus is to: A: provide oxygen and other nutrients to the fetus. B: dilate and expel the baby from the cervix. C: provide a cushion and protect the fetus from infection. D: house the fetus as it grows for 40 weeks.

*D. house the fetus as it grows for 40 weeks.* The uterus is a muscular organ where the fetus grows for 37 to 42 weeks (average of 40 weeks). It is responsible for contracting during labor, which in conjunction with dilation of the cervix (the opening of the uterus), expels the baby from the uterus into the birth canal. The placenta is the organ of exchange that delivers oxygen and other nutrients from the mother to the fetus and returns metabolic waste products from the fetus to the mother. The amniotic sac, also called the bag of waters, provides a cushion for the developing fetus and helps protect it from infection.

As an EMT, your primary responsibility is to: A: provide competent patient care. B: ensure the safety of your partner. C: transport all patients to the hospital. D: keep yourself as safe as possible.

*D. keep yourself as safe as possible.* As an EMT, your primary responsibility is to yourself. An injured or dead EMT is of no use to a patient. After ensuring the safety of yourself, your crew, and any bystanders, patient care should be initiated.

A 22-year-old female woman was shot by her husband. Law enforcement is at the scene and has the husband in custody. The patient is conscious, but extremely restless, and is pale and diaphoretic. As your partner administers high-flow oxygen, you should: A: take her BP to detect hypotension. B: keep her warm by applying blankets. C: compare her carotid and radial pulses. D: look for and control any bleeding.

*D. look for and control any bleeding.* The primary assessment of any patient includes ensuring a patent airway, assessing breathing adequacy, administering high-flow oxygen or assisting ventilations, assessing circulation, and controlling all active bleeding. You and your partner must work as a team; as your partner administers high-flow oxygen, you should be looking for her gunshot wound(s) and ensuring that all bleeding is controlled. After the primary assessment and management, begin treating her for shock (eg, applying a blanket, elevating her lower extremities [if local protocol permits]) and perform a rapid head-to-toe assessment to search for other injuries that may not have been obvious during the primary assessment. Assess the patient's vital signs after all life-threatening injuries or conditions have been identified and corrected.

A middle-aged man was found floating facedown in a small pond. When you arrive at the scene, bystanders are present, but nobody has removed him from the water because they thought he was dead. After reaching the victim, you should: A: float a buoyant backboard under him, remove him from the pond, and begin rescue breathing. B: stabilize his head as you remove it from the water and open his airway by tilting his head back. C: grab him by his clothing, remove him from the pond, and assess for breathing and a pulse. D: move him to a supine position by rotating the entire upper half of his body as a single unit.

*D. move him to a supine position by rotating the entire upper half of his body as a single unit.* After safely reaching a drowning victim, you should first turn him to a supine position by rotating the entire upper half of the body as a single unit; protect the cervical spine as you do this because a spinal injury cannot be ruled out. Open the patient's airway, without manipulating the neck, and begin rescue breathing while still in the water. Float a buoyant backboard under the patient, secure him to it, and remove him from the water. After removing the victim from the water, assess for a pulse. If the victim is pulseless, begin CPR, dry him off, and apply the AED as soon as possible.

The middle, muscular layer of the heart is called the: A: endocardium. B: pericardium. C: epicardium. D: myocardium.

*D. myocardium.* The heart has four layers. The inner layer is called the endocardium, the middle layer is composed of muscle and is called the myocardium (myo = muscle), and the outer layer of the heart itself is called the epicardium. The pericardium, which is a thin, fibrous membrane, encapsulates the entire heart.

When attaching an oxygen regulator to a D cylinder and preparing it for use, you should recall that: A: the cylinder must remain in a standing position at all times or it will not deliver any oxygen. B: the cylinder should be taken out of service and refilled when it contains less than 750 psi. C: a pressure-compensated flowmeter should be used when lying the oxygen cylinder down. D: oxygen supports combustion and should not be used where sparks are easily generated.

*D. oxygen supports combustion and should not be used where sparks are easily generated.* Oxygen does not burn or explode; however, it does support combustion. A small spark, even a lit cigarette, can become a flame in an oxygen-rich atmosphere. Therefore, you must ensure that the environment in which you will use oxygen is adequately ventilated, especially in industrial settings where hazardous materials may be present and where sparks are easily generated. Never leave an oxygen cylinder standing unattended; the cylinder can be knocked over, injuring the patient or damaging the equipment. The D cylinder (small cylinder carried to the patient) should be taken out of service and refilled when the pressure inside it falls below 500 psi. The pressure-compensated flowmeter, which contains a ball and float that rises or falls according to the gas flow, can only be used when an oxygen cylinder is upright, which is why it is used with on-board oxygen (M cylinder). The Bourdon-gauge flowmeter does not require the oxygen cylinder to be upright, which is why it is used with D cylinders.

A 21-year-old male was bitten on the left forearm by a dog. He is conscious and alert and denies any other injuries. An animal control officer is at the scene and has contained the dog. Your assessment of the patient's arm reveals a large avulsion with a peeled back flap of skin. Distal circulation is adequate and the patient is able to feel and move his fingers. In addition to bleeding control, you should: A: irrigate the wound for at least 15 minutes. B: apply oxygen via a nonrebreathing mask. C: perform a rapid head-to-toe assessment. D: replace the avulsed flap to its original position.

*D. replace the avulsed flap to its original position.* An avulsion is an injury that separates various layers of soft tissue, usually between the subcutaneous layer and fascia, so that they become completely detached or hang as a flap. The patient's injury is isolated and not life-threatening; therefore, a rapid head-to-toe assessment is not indicated. Unless your protocols specify otherwise, oxygen is usually not necessary for patients with isolated, non-life-threatening soft tissue injuries. If the avulsed tissue is hanging from a small piece of skin, circulation through the flap may be at risk. If you can, replace the avulsed flap to its original position, as long as it is not visibly contaminated with dirt and/or other foreign materials, and then cover the wound with a dry sterile dressing. Unless the wound is grossly contaminated with dirt or debris, irrigation is usually deferred until the patient is evaluated by a physician. Furthermore, flushing an open wound may force dirt or other debris into the wound, increasing the risk of infection.

You should not attempt to actively rewarm a patient with moderate or severe hypothermia in the field because: A: the risk of inadvertently inducing hyperthermia is too high. B: active rewarming has been shown to cause severe hypertension. C: it is painful for the patient and you cannot give analgesic drugs. D: rewarming too quickly can cause a fatal cardiac dysrhythmia.

*D. rewarming too quickly can cause a fatal cardiac dysrhythmia.* When caring for a patient with hypothermia, your goal is to prevent further heat loss; this involves removing wet clothing, applying warm blankets, and allowing the patient's body temperature to rise gradually and naturally (passive rewarming). If the patient is moderately or severely hypothermic, you should not try to rewarm him or her actively (placing heat on or into the body). Rewarming too quickly may cause a fatal cardiac dysrhythmia, such as ventricular fibrillation (V-Fib). Active rewarming may also cause rewarming shock, a condition in which the blood vessels dilate when heat is applied to the body, resulting in significant hypotension. For these reasons, active rewarming should only be performed in the controlled setting of a hospital.

When ventilating an apneic patient, you note decreased ventilatory compliance. This means that: A: fluid is occupying the alveoli. B: the upper airway is blocked. C: you meet no resistance when ventilating. D: the lungs are difficult to ventilate.

*D. the lungs are difficult to ventilate.* As it applies to artificial ventilation, compliance is the ability of the lungs to expand during ventilation. Increased ventilatory compliance means that no resistance is met when you ventilate the patient; you can ventilate the lungs with ease. Decreased ventilatory compliance means that significant resistance is met when you ventilate the patient; the lungs are difficult to ventilate. Conditions such as upper airway obstruction, widespread bronchospasm, fluid in the alveoli (eg, pulmonary edema), and COPD can all cause decreased ventilatory compliance.

While functioning at a large-scale terrorist incident, it is important for the EMT to: A: begin immediate treatment of the most critically injured. B: identify the person or persons responsible for the event. C: avoid placing any casualty in a "delayed" treatment status. D: use triage and base patient care on available resources.

*D. use triage and base patient care on available resources.* During a terrorist incident, the basic foundations of triage and patient care remain the same; however, the treatment can and will vary. Terrorist incidents can produce a single casualty, hundreds of casualties, or thousands of casualties. When presented with wide-spread mass casualties, you must remember situational awareness. What you do in one situation may not be appropriate for another situation. In large-scale terrorist incidents, it is important to use triage and base patient care on available resources. When triaging casualties, use the same triage process that you would for any other mass-casualty incident. Remain focused on providing the greatest good for the greatest number of people, not the person or persons responsible for the incident.


Related study sets

Gastrointestinal Medication (Chapter 53)

View Set

Principles of Marketing - Exam 1

View Set

Arrays/Matrices Practice Questions - Java

View Set

Java Quiz 7 - Classes & Functions

View Set